solve this equation by using the look inside method to rewrite the problem

-81 = 3(5 - 4r)

Answers

Answer 1
-81=3(5-4r)
27=-(5-4r)
27=-5+4r
-4r=-5-27
-4r=-32
r=8

Hope my answer helped u :)

Related Questions

Is 0.875 irrational or rational and why?

Answers

Yes it’s rational It can be expressed as the quotient of two integers

tìm sự hội tụ của

∑ [tex]\frac{4^{n} }{n!}[/tex]
n=1

Answers

Answer:

100002

Step-by-step explanation:

bi the react

alexandra already has 10 plants in her backyard and she also can grow 2 plants with every seed packet she uses .with 14 seeds packets how many total plants can alexandra have in her backyard

Answers

2x14=28 +10=38 plants in all

Emergency funds are important because they

Answers

Answer:

an emergency fund allows you to live for a few months if you lose your job or if something unexpected comes up that costs a fair chunk of money to cover

Answer:

If there ever was an emergency you will need money to pay/afford the injury

Step-by-step explanation:

Evaluate the expression x²+yz for x=-3, y=5, and z=2

Answers

Answer:

I think that the answer is 19.

Step-by-step explanation:

Writing out the problem

x²+yz

Substituting the given values of the variables

(-3)²+(5)(2)

Simplifying (-3)², which is 9

9+(5)(2)

5 times 2 equals 10

9+10

19

I hope this helped you. Enjoy your day, and take care :)

Consider the function below.
g(x) = 2x - 1


Find the difference quotient below (where h = 0) and simplify your answer.

g(x + h) - g(x) / h

Answers

Answer:

Step-by-step explanation:

Which of the following is the correct way to express 320 in scientific notation?
A. 3.2 x 10²
B. 32.0 x 10²
C. 3.2 x 10³
D. 0.32 x 10²

Answers

Answer:

Hi, there!

Option A. 3.2 x 10²

Step-by-step explanation:

320 written in scientific notation is 3.2 × 102.

Write 9 1/100 as a decimal number

Answers

Answer:

9.01

Step-by-step explanation:

9 1/100 = 9 + 1/100

1/100= 1 divded by 100

=9+0.01=9.01

the value of 9 1/00 in decimal, is 9.01

What is Simplification?

Simplifying procedures is one way to achieve uniformity in job efforts, expenses, and time. It reduces diversity and variation that is pointless, harmful, or unneeded. Parenthesis, exponents, multiplication, division, addition, and subtraction are all referred to as PEMDAS. The order of the letters in PEMDAS informs you what to calculate first, second, third, and so on, until the computation is finished, given two or more operations in a single statement.

Given a number 9 1/100

in decimal, we can also write it as

9 1/100 = 9 +1/100

=> 9 1/100 = 9.01

therefore, in decimal the value of 9 1/100 is 9.01.

Learn more about Simplification here:

https://brainly.com/question/28996879

#SPJ

Find the probability that a student earns a grade of A,B, or C
P=

Answers

Answer:

Step-by-step explanation:

Answer: 0.86

Step-by-step explanation:

please help me out on this question.

Answers

Answer:

Aaron: Keep the base the same. Add -5 and -2 and the result is a new exponen. Next simplify 3^-7. The simplified form is 1/2187.

What is 2x+2y=8 in standard form

Answers

Answer:

x+y=4

Step-by-step explanation:

2x+2y=8

By taking out 2 from 2x+2y as common terms, we get

2(x+y)=8

Now, by dividing both sides by 2, we get

2(x+y)/2=8)2

x+y=4

9. Use Structure Point K is of the way from |(4,-5) to L(0, -7). a. What are the coordinates of Kif n = 4? b. What is a formula for the coordinates of K for any n?​

Answers

Dividing the segments, it is found that:

a) The coordinates of K if n = 4 are [tex](5,-frac{9}{2})[/tex]b) [tex]K - J = \frac{1}{n}(J - L)[/tex]

----------------------------

The endpoints are [tex]J(4,-5)[/tex] and [tex]L(0,-7)[/tex].Point K is (x,y).Since point K is [tex]\mathbf{\frac{1}{n}}[/tex] of the way from J to L, it means that:

[tex]K - J = \frac{1}{n}(J - L)[/tex]

And this is the answer to option b.

----------------------------

Option a:

If [tex]\mathbf{n = 4}[/tex], it has that:

[tex]K - J = \frac{1}{4}(J - L)[/tex]

This is applied to find both the x-coordinate and the y-coordinate of K.

----------------------------

The x-coordinate of K is x.The x-coordinate of J is 4.The x-coordinate of L is 0.

Thus:

[tex]K - J = \frac{1}{4}(J - L)[/tex]

[tex]x - 4 = \frac{1}{4}(4 - 0)[/tex]

[tex]x - 4 = 1[/tex]

[tex]x = 5[/tex]

----------------------------

The y-coordinate of K is y.The y-coordinate of J is -5.The y-coordinate of L is -7.

Thus:

[tex]K - J = \frac{1}{4}(J - L)[/tex]

[tex]y - (-5) = \frac{1}{4}(-5 - (-7))[/tex]

[tex]y + 5 = \frac{2}{4}[/tex]

[tex]y = \frac{1}{2} - 5[/tex]

[tex]y = \frac{1}{2} - \frac{10}[2}[/tex]

[tex]y = -\frac{9}{2}[/tex]

The coordinates of K if n = 4 are [tex](5,-frac{9}{2})[/tex]

A similar problem is given at https://brainly.com/question/20847604

Y/3 -4 = 2 What does Y equal?

Answers

Answer:

y = 18

Step-by-step explanation:

y/3 - 4 = 2

y/3 = 6

y = 18

Please Help! I will give you a lot of points! You don't have to show your work unless you want to. Just the answer.

Answers

Answer:

52

Step-by-step explanation:

15+7+19+11+0=52

may help.

Convert into numbers

[tex]\\ \sf\longmapsto 15+7+19+13+11+0[/tex]

[tex]\\ \sf\longmapsto 22+32+11[/tex]

[tex]\\ \sf\longmapsto 55+11[/tex]

[tex]\\ \sf\longmapsto 66[/tex]

write an expression that removes any perfect square factors in the radicand. Assume all variables are positive.

Answers

X=2.x is wrong use Photo math

A regular hexagon is a six sided figure with all sides equal and all six angles equal. Find the length of the sides if the perimeter of the regular hexagon is 396 yd.

Answers

The answer is 66 because 369/6 is 66

Mack beat his opponent by
SEVEn points in a game which
ends when a total of
101 points have been scored.
How many point did Mark
have​

Answers

Answer:

94

Step-by-step explanation:

When a player reaches 101 points, they win. So that means that Mack reached 101 points. Since his opponent was seven points behind, we can subtract 7 from 101 to find Mark's, that is Mack's opponent's, score:

[tex]101-7=94[/tex]

Therefore, we can safely conjecture that Mark scored 94 points.

Solve for X. Will give a lot of points. HELP!!!

Answers

Answer:

7).

RT = RS + ST

[tex]10 = (2x - 20) + (2x - 18) \\ 10 = (2x + 2x) + ( - 20 - 18) \\ 10 = 4x - 38 \\ 4x = 10 + 38 \\ 4x = 48 \\ { \boxed{x = 12}}[/tex]

8).

LN = LM + MN

[tex](5x - 2) = 12 + (x + 2) \\ (5x - x) = 12 + 2 + 2 \\ 4x = 16 \\ { \boxed{x = 4}}[/tex]

I hope this helps. I wasn’t sure how to put it into words.

Find the inverse of the function: f(x)=3x−6

A f−1(x)=−2/3x
B f−1(x)=1/3x+2
C f−1(x)=−1/3x+12

Answers

Answer:

B. [tex] {f}^{ - 1} (x) = \frac{1}{3} x + 2[/tex]

Step-by-step explanation:

[tex]f(x) = 3x - 6[/tex]

[tex]y = 3x - 6[/tex]

[tex]x = 3y - 6[/tex]

[tex]3y - 6 = x[/tex]

[tex]3y = x + 6[/tex]

[tex]y = \frac{1}{3} x + 2[/tex]

[tex]\boxed{\green{ {f}^{ - 1} (x) = \frac{1}{3} x + 2}}[/tex]

Katrina said the number
0.101100111000... is a rational number because it consists
only of the digits 0 and 1, and these digits repeat. Do you
agree or disagree? Explain.

Answers

The pattern does NOT repeat. The number of 1’s and 0’s are increasing.
A longer version is 0.101100111000111100001111100000111111000000...
The number is irrational.

help please !
Enter your answer and show all the steps that you use to solve this problem in the space provided Simplify the expression (7 + 5) +4.13 - 2

please show me the steps to get the answer ​

Answers

Answer:

14.13

Step-by-step explanation:

(7 + 5) +4.13 - 2

PEMDAS says parentheses first

(12) +4.13 - 2

Then add and subtract from left to right

16.13 -2

14.13

0.189 divided by 1.75=
Explain how you get this answer

Answers

9514 1404 393

Answer:

  = 0.108

Step-by-step explanation:

Use a calculator or long division.

When using long division, it is convenient to move the decimal points in both numbers to the right 2 places. This makes the divisor an integer, so makes it easier to keep track of appropriate place value in the quotient.

  0.189/1.75 = 18.9/175 = 0.108

The average yearly cumulative rainfall in Tucson by month is shown in the graph below. Does this
graph represent cumulative rainfall as a function of the day? Why or why not? What is the average
annual cumulative rainfall in Tucson? What is the approximate average rainfall in December? During
which month is average rainfall the greatest? How do you know?

Answers

Answer:

The average annual cumulative rainfall is  [tex]1[/tex] inch per month.

During month of August average rain fall is greatest because slope in this month is maximum.

Step-by-step explanation:

The graph does not represent cumulative rainfall as a function of day.

Because they have average all values for month Total cumulative rainfall is [tex]12[/tex] inches.

Total period is [tex]12[/tex] month

So average annual cumulative rainfall is

[tex]$12 / 12=1$[/tex] inch per month

Average rain fall in month of December= (average cumulative rainfall in December[tex])-([/tex]average  cumulative rainfall in November)

[tex]=12-11[/tex]

[tex]$=1$[/tex] inch

During month of August average rain fall is greatest because slope in this month is maximum.

Learn more about average, refer :

https://brainly.com/question/22390452

How can you solve
an equation or inequality by graphing?

Answers

Example: y ≤ x + 2

There are three steps:

Rearrange the equation so "y" is on the left and everything else on the right.
Plot the "y=" line (make it a solid line for y≤ or y≥, and a dashed line for y< or y>)
Shade above the line for a "greater than" (y> or y≥)
or below the line for a "less than" (y< or y≤).

What is -1 7/8 + -2 1/4? Please explain why and a step by step :).

Answers

9514 1404 393

Answer:

  -4 1/8

Step-by-step explanation:

Many people find it easier to factor out -1, then do the addition. The fractions need a common denominator before they can be added. A suitable denominator that is a multiple of both 4 and 8 is 8.

The fraction 1/4 = (1/4)(2/2) = (1·2)/(4·2) = 2/8.

__

So, the problem can be rewritten as ...

  -(1 7/8 +2 2/8)

  = -((1+2) +(7/8 +2/8))

  = -(3 +9/8)

The fraction 9/8 is an improper fraction equal to 8/8 +1/8 = 1 1/8. Then the sum is ...

  -(3 +1 1/8) = -4 1/8

Write an expression for the operation described below.


multiply 6 by u

Answers

Answer:

6u

Step-by-step explanation:

it is the algebraic answer.

hope this helps!

Answer:

the answer is 6×u

Step-by-step explanation:

by and multiply are multiplication

2.
The perimeter of a rectangle is 82 meters. If the length of the rectangle is 20 meters more than the width
of the rectangle, find its length and width.

Answers

Answer:

30.5 meters

10.5 meters

Step-by-step explanation:

x = length

y = width

x = y + 20

2x + 2y = 82 (the perimeter of a rectangle)

using the first equation as substitute in the second equation we get

2×(y + 20) + 2y = 82

2y + 40 + 2y = 82

4y = 42

y = 42/4 = 21/2 = 10.5 meters

x = y + 20 = 21/2 + 20 = 21/2 + 40/2 = 61/2 = 30.5 meters

2. Describe a situation that could be modeled with the ratio 4: 1.​

Answers

Answer:

u can do something like u have 8 pencils and 2 pens

Step-by-step explanation:

cause two divided by two is four and u only have to divide it once so that how u get that answer

Write the following as an algebraic expression. Simplify if possible.
Add 9x - 8 to 4x - 1.

Answers

Answer:

13x -9

Step-by-step explanation:

4x-1 + 9x-8

Combine like terms

4x +9x  -1-8

13x -9

Answer:

9x+−8+4x+−1

(9x+4x)+(−8+−1)

13x+−9

Step-by-step explanation:

so sorry if wrong

Solve the equation z^4+4z^2+16=0

Answers

Answer:

As this is a quartic equation, we should expect four roots (although some may end being repeated). Notice also that the equation is symmetrical, which suggests that there might be an element of symmetry in the answer.

First, as this equation is a quadratic in terms of z2 , we can use our formula for finding the roots of a quadratic equation to give the value(s) of z2 . [If you have a problem seeing this, use the substitution y=z2 , and you get a quadratic in y .]

z2=−4±42−4×1×16√2×1=−4±16−64√2

=−4±−48√2=−2±12−−√i=−2±23–√i

=4(−12±3√2)i

=4(cos(2kπ3)+isin(2kπ3)) , where k∈1,2

=4e2kiπ3

As this is the value of z2 , whereas we want z , we need to take the square roots of both sides

z=4–√ekiπ3+nπ , where n∈0,1

=2e(k+3n)iπ3

=2cis((k+3n)iπ3)

As we have two values of k and two values of n , this gives us four solutions.

Writing these angles in terms of degrees, we have: 60, 120, 240 and 300

Evaluating, we end up with the four solutions

z=1±3–√i∪z=−1±3–√i

You can substitute z^2 = x,

Then you get x^2+4x+16=0

Step-by-step explanation:

z=1±3–√i∪z=−1±3–√i

You can substitute z^2 = x,

Then you get x^2+4x+16=0

Then you can use classical x1,2 = (-b +- √[b^2–4ac])/2a

X1,2=(-4+-√[16–16*4])/2

After you solve that you substitute back z^2 = x

Other Questions
where is the headquarters of the general assembly A noun shows action or a state of being true or false liam had a container of oatmeal that contained 5 1/2 cups of oatmeal. Liam ate 4/9 cup of oatmeal every morning before school for a week. How many cups of oatmeal that Liam had left in the container at the end of the five-day week Locate the object pronoun in this sentence.My friends invited me for dinner at their lake house.Click the answer you think is right.mymetheirfor Given that ()=2(3)28f(x)=2(x3)28, determine the true statement.The x intercepts of the graph of f(x) are (1,0) and (5,0).Group of answer choicesThe range of f(x) is8y8.The graph of f(x) is decreasing from(3,)(3,).The graph of f(x) has a maximum point at (3, -8).The domain of f(x) is3x3 In the second paragraph, you need to explain the role of citizens in your country. The country is north Korea a:b=2:3b:c= 5:6Show that a:c=5:9 Which of the following descriptions corresponds with conservation biology?a.)Has the intention of repairing damaged or destroyed ecosystemsb.)Protection and management of natural resourcesc.)Consideration of both the living and non-living components of an ecosystemd.)Has the intention of protecting biodiversity 2x^2-y if x=3 and y=8 Can someone please help me please? The output signal from an analogue sensor is sampled every 195 s to convert it into a digital representation. What is the corresponding sampling rate expressed in kHz?According to the Sampling Theorem, for this sampling rate value, what approximately could be the highest frequency present in the signal, in kHz, assuming the lowest frequency is very close to zero?If each sample is now quantised into 512 levels, what will be the resulting bitrate in kbps?Give your answer in scientific notation to 2 decimal places.Hint: you need to determine the number of bits per sample that produces 512 quantisation levels. 6-5(2x-3)=4x+7Solve for x Charles delivers flowers. He aerns $5.57 to deliever a flower arrangement. Last week he made $746.38. How many arrangements did he deliver? Sandy can read 25 words in 5 seconds. At thatrate Write an essay in about "Importance of Education in human life" Was absent and need help ASAPWhich of the following foods contain protein? More than one answer may be correct. a) Honey b) Egg white c) Canola oil d) Corn Starch e) Potato f) Apple Juice g) Water A single element of culture is called what Find the x-intercept and y-intercept for the graph of 27x - 18y = 54A. x-intercept = 0.5, y-intercept = 0.3 B. x-intercept = 2, y-intercept = -3 C. x-intercept = 3, y-intercept = -2 D. x-intercept = 4, y-intercept = 3 What is the solutiony=-x+2y=-3x-2 Find the measures of the indicated angles pictures included!!!